Endocrine Practice Questions

Lakukan tugas rumah & ujian kamu dengan baik sekarang menggunakan Quizwiz!

A new patient is seen in your internal medicine office today. She is coming in to request the removal of several skin tags. She is a 55-year-old woman with a history of untreated acromegaly. A health maintenance plan is set up with the patient, and includes a colonoscopy. This patient is at increased risk for which of the following findings on colonoscopy? A Anal fissures B Ulcerative colitis C Colon polyps D Pseudomembranous colitis E Colonic fistulas

C The correct choice is C, colon polyps. Approximately 30% of patients with acromegaly have been found to have colon polyps. These patients also have an increased risk of colon cancer. Patients with acromegaly have not been found to be at increased risk for the other response choices listed here.

Which of the following cardiac medications is known to cause clinically significant hypothyroidism? A Furosemide B Captopril C Amiodarone D Digoxin E Dopamine

C The correct choice is C; Amiodarone is an antiarrythmic medication used to treat patients with recurrent ventricular tachycardia or fibrillation. Its structure is similar to thyroid hormone, and its metabolites antagonize thyroid hormone function in approximately 13% of patients treated with amiodrone in the United States. Choice A, furosemide, choice B, captopril, choice D, digoxin, and choice E, dopamine, have no effect on thyroid function.

Release of which of the following substances is triggered by pituitary growth hormone and promotes growth of other tissues in the body? A C-pepide B IL-I C IGF-I D Thyroxine E Catecholamines

C The correct choice is C, IGF-I or insulin like growth factor I. This growth factor leads to increased DNA, RNA, and protein synthesis, which leads to overgrowth of bone, soft tissue, and cartilage. Choice A, c-peptide, is a part of the prohormone of insulin. Choice B, IL-I or interleukin I, is an important cytokine that promotes cell activation. Choice D, thyroxine, potentiates the actions of growth hormone on tissues.

ou are reviewing the labs of a 78-year-old male with a 4-month history of fatigue. You suspect primary hypothyroidism as your diagnosis. What laboratory findings would support your conclusions? A elevated thyroid-stimulating hormone (TSH) B low thyroid-stimulating hormone (TSH) C low parathyroid hormone (PTH) D high total triiodothyronine (T3) E high free triiodothyronine (FT4)

A The correct answer is (A). An elevated TSH and low FT 4 is characteristic of primary hypothyroidism. A low TSH and elevated T 3 or FT 4 would suggest hyperthyroidism. Low PTH suggests hypoparathyroidism.

A 55-year-old female presents to the emergency department with acute tetany and carpal pedal spasms. She has a positive Trousseau's phenomenon on exam. Her calcium and PTH is low. What is the most appropriate treatment at this time? A calcium gluconate IV B thiamine C magnesium oxide D vitamin D E calcium carbonate

A The correct answer is (A). The patient has symptoms of acute hypocalcemia of hypoparathyroidism, with a low calcium and PTH. Acute treatment of hypocalcemic tetany due to hypoparathyroidism is calcium gluconate IV to maintain the calcium between 8-9 mg/dl. Once the patient is stable p.o. calcium such as calcium carbonate and p.o. vitamin D is added. If magnesium is low, it should also be corrected using magnesium sulfate IV. Once corrected, the patient may need to continue supplementation with p.o. magnesium oxide.

A 55-year-old patient presents with symptoms of polyuria and is worried that he may have diabetes mellitus. His past medical history includes hypertension and lung cancer. His family history is negative for diabetes mellitus. He works as an accountant and has no history of head trauma. He has an average diet and fluid intake. His fasting plasma glucose is 110 mg/dL and his hemoglobin A1c is 5%. What is the most likely cause of his polyuria? A Cancer related diabetes insipidus B Type 2 diabetes mellitus C Nephrogenic diabetes insipidus D Familial hypothalamic diabetes insipidus E Primary polydipsia

A The correct choice is A, cancer related diabetes insipidus. Solid tumors, such as cranipharyngiomas and metastatic lung and breast cancer, are known to cause diabetes insipidus. This patient has a history of lung cancer.

A 1-year-old adopted Chinese infant is brought into the pediatrician's office for a routine visit. The parents are concerned because they think the child is short for her age. They do not have very much information relating to the birth of the child, other than that she was considered slightly short at birth and she had a seizure at one month of age. The foster parents of the child were not concerned, since the infant was somewhat chubby and ate well. She has no history of hypotonia. Upon exam you note normal shaped eyes with mild nystagmus. The infant is less than the third percentile for weight and her limbs are in normal proportion to her height. Her lung and heart exam are normal. CBC is normal. What is the most likely cause of her short stature? A Congenital growth hormone deficiency B Sickle cell anemia C Prader-Willi syndrome D Osteochondrodysplasia E Cystic fibrosis

A The correct choice is A, congenital growth hormone deficiency. Parents of children with this disorder typically become concerned when the child is between one and two years of age. This patient fits the characteristic picture of this disorder, with short stature, increased fat mass, and hypoglycemia due to relatively unopposed insulin action. This may have been the cause of her seizure. She may also have optic hypodysplasia as a cause of her nystagmus. Choice B, sickle cell anemia, can be associated with short stature. However, the CBC from this patient was normal. Choice C, Prader-Willi syndrome, is also associated with short stature. Children with this disorder commonly have almond shaped eyes and hypotonia at birth. Choice D, ostrochondrodysplasia, is a type of inherited skeletal abnormality that presents with short stature and abnormal body proportions. This patient doesn't have abnormal body proportions. Choice E, cystic fibrosis, can be associated with growth abnormalities, but this patient has no evidence of lung disease.

A 41-year-old female comes in for her annual exam. She generally feels well and has no complaints other than some general fatigue. On examination you note a normal size thyroid with a palpable firm 1.5 cm nodule in the left lobe of her thyroid. What is the best test to order that will provide a definitive diagnosis? A thyroid ultrasound B thyroid uptake scan C fine needle biopsy of nodule D MRI thyroid E TSH

C The correct answer is (C). The patient has a solitary, > 1cm, firm nodule, which is suspicious for a thyroid malignancy. A fine needle biopsy of the nodule would provide you with a definitive diagnosis. A thyroid ultrasound, thyroid uptake scan, and TSH would be helpful in determining the etiology of the nodule but will not provide you with a definitive diagnosis. Since this patient has a suspicious clinical examination a biopsy is indicated. MRI of the thyroid is not a preferred test.

40-year-old female presents to your office with symptoms of weight gain, hirsuitism, and easy bruising. Past medical and surgical history is noncontributory. She drinks one glass of wine on weekends and does not smoke cigarettes. She takes one multivitamin daily. Upon physical exam, you note facial fullness, central obesity, and thin skin. Which of the following is a valuable biochemical screening test for this patient that will aide in the diagnosis? A Dexamethasone suppression test B Radioactive iodine uptake C Glucose tolerance test D Cosyntropin stimulating test E Plasma fractionated free metanephrines

A The correct choice is A, dexamethasone suppression test. This patient is presenting with classic signs and symptoms of Cushing's syndrome. The dexamethasone suppression test is a simple test of the hypothalamic-pituitary-adrenal axis, and requires ingestion of oral dexamethasone at nighttime and a blood test in the morning hour, to measure the amount of plasma cortisol. Most patients with Cushing's syndrome demonstrate a lack of normal axis suppression and present with a morning plasma cortisol level >5 mcg/dL.

Which of the following sets of lab values is most consistent with the diagnosis of Hashimoto's thyroiditis? A High serum TSH, low serum total T4, and high thyroidal peroxidase antibodies B Low serum TRH, low serum TSH, and low serum free T3 levels C Normal TSH, normal serum total T4, and normal radioactive iodine uptake D Low serum TSH, high serum total T4, and high thyroid stimulating antibodies E High serum TSH, high radioactive iodine uptake, and high serum free T3

A The correct choice is A, high serum TSH, low serum total T 4 , and high thyroidal peroxidase antibodies. Hashimoto's thyroiditis is the most common cause of primary hypothyroidism and is autoimmune in nature. The serum thyroid hormone levels are low, secondary to the destruction occurring in the thyroid gland. The negative feedback loop causes the pituitary to respond by increasing production and secretion of TSH. Thyroidal peroxidase, thyroglobulin, and TSH receptor blocking autoantibodies can be found in these patients.

Which of the following types of thyroid cancer involve the parafollicular cells (or C cells) of the thyroid? A Medullary B Papillary C Follicular D Lymphoma E Anaplastic

A The correct choice is A, medullary thyroid carcinoma. Unlike papillary (choice B) and follicular (choice C) thyroid cancers, which arise from thyroid epithelial cells, medullary thyroid cancer arises from the parafollicular cells of the thyroid. Anaplastic thyroid cancer, choice E, arises from undifferentiated cells. Thyroid lymphoma, choice D, is most commonly of B cell follicle center cell origin.

A patient was prompted to visit his health care provider after his wife started to notice that he was not interested in eating, has lost weight, and has been suffering from nausea for the last few weeks. The practitioner notes hyperpigmentation of the patient's skin, although the patient denies any recent sun or tanning salon exposure. Routine non-fasting blood work reveals the following: Sodium = 130 meq/L Potassium = 5.2 meq/L Chloride = 105 meq/L Glucose = 135 mg/dL Hemoglobin = 13.0 g/dL Hematocrit = 39.0 WBC count = 8,000/mm 3 Which of the following physical exam findings would you expect to see in this patient? A Orthostatic hypotension B Wide, purple striae C Central obesity D Full facial features E Exophthalmous

A The correct choice is A, orthostatic hypotension. The first step in the discussion of this patient is the suspected diagnosis of adrenal insufficiency. Patients with this disorder will have an excess of ACTH, which will act like melanocyte stimulating factor on the skin and cause hyperpigmentation. In adrenal insufficiency, aldosterone is deficient, thereby causing a decrease in sodium retention and potassium excretion. Hypotension is found in approximately 90% of these patients, sometimes associated with syncope as well. Choices B, C, and D are found in patients with cortisol excess. Choice E can be found in patients with Graves' disease.

A patient presents with a several year history of developing coarse facial features, prognathism, widely spaced teeth, and sleep apnea. Upon exam, you also note oily skin and high blood pressure. What is the most likely cause of this patient's symptoms? A Pituitary adenoma B Pheochromocytoma C GHRH secreting hypothalamic tumor D Adrenal carcinoma E Ectopic ACTH secretion

A The correct choice is A, pituitary adenoma. The patient is presenting with classic signs and symptoms of acromegaly. This disorder is caused most commonly by a growth hormone secreting pituitary adenoma. On rare occasions, it has been caused by choice C, a GHRH secreting hypothalamic tumor.

A 40-year-old female presents to your office with symptoms of weight gain, hirsuitism, and easy bruising. Past medical and surgical history is noncontributory. She drinks one glass of wine on weekends and does not smoke cigarettes. She takes one multivitamin daily. Upon physical exam, you note facial fullness, central obesity, and thin skin. Which of the following is the most common cause of her symptoms? A Pituitary adenoma B Iatrogenic C C Adrenal micronodular hyperplasia D Adrenocortical adenoma E Ectopic ACTH syndrome

A The correct choice is A, pituitary adenoma. This patient's clinical presentation is typical in Cushing's syndrome. The most common cause of Cushing's syndrome (other than ingestion of oral steroid medications) is Cushing's disease. This disease is caused by a benign, ACTH secreting pituitary adenoma. Choice B, iatrogenic, refers to the ingestion of prescribed (or non-prescribed) oral corticosteroid medications. This is frequently seen in patients requiring long-term oral steroid medications. This patient does not have this type of history. Choice C, adrenal micronodular hyperplasia, and choice D, adrenocortical adenoma, can cause Cushing's syndrome at less frequent incidence. Choice E, ectopic ACTH syndrome, presents more commonly in males with extremely elevated levels of plasma cortical and ACTH. These patients commonly have a positive history of an ectopic source of the ACTH, such as in small cell carcinoma of the lung.

A young man presents with difficulty breathing at times. Upon exam you note evidence of a firm, fixed thyroid nodule with extension toward the trachea and surrounding muscles. The patient has a family history of thyroid cancer. You are concerned that the patient may have medullary thyroid cancer. Which of the following lab tests would you monitor in this patient after treatment? A Serum calcitonin B Plasma alkaline phosphatase C Serum anti-thyroglobulin antibodies D Serum T3 resin uptake E Urine CA-125 levels

A The correct choice is A, serum calcitonin. Both calcitonin and CEA are secreted by medullary thyroid cancer cells, and are used both in diagnosis and monitoring of patients after treatment. Serum CA-125 has been used to investigate and follow patients with malignancies, such as ovarian cancer.

A 60-year-old female presents to her primary care practitioner for her yearly check up. Her past medical history is significant only for a history of repeat kidney stones. She enjoys gardening outside, has no significant family history, and is not taking any medications. Routine mammograms have been normal. During the review of her systems, the patient describes feeling tired lately and has noticed muscle aches over the last few weeks. Initial lab values are shown below: Hemoglobin= 12.0 gm/dL Hematocrit = 36% BUN= 18 mg/dL Creatinine= 1.0 mg/dL Calcium= 12 mg/dL Phosphate= 2.0 mg/dL Intact PTH= 80 pg/mL TSH= 3.0 uU/L What is the most likely pathology associated with the findings in this patient? A Solitary parathyroid adenoma B Excessive Ingestion of calcium C Parathyroid carcinoma D Medullary thyroid cancer E Chronic renal disease

A The correct choice is A, solitary parathyroid adenoma. Primary hyperparathyroidism is caused by a solitary adenoma in 80% of patient cases. Choices B, D, and E do not cause primary hyperparathyroidism. Choice C can cause primary hyperparathyroidism, but with an incidence of less than 1%.

A 72-year-old female is a new patient in your office who is not taking any medications other than cranberry pills. She has been healthy most of her life but recently has had several episodes of kidney stones. Furthermore, in the past 6 months she has felt somewhat tired and depressed but can't figure out why. She had a bone density during a health screening and was told she has osteoporosis but is not taking anything for this at this time. She wonders if osteoporosis would cause her joint and bone pains. She has also had some abdominal discomfort and constipation recently, which does not seem to be attributed to what she eats. Her labs in the office today reveal serum calcium of 11.4 mg/dl with normal renal function. What is the most likely cause of her symptoms? A Cushing's syndrome B hyperparathyroidism C hypothyroidism D hypoparathyroidism E hypopituitarism

B The correct answer is (B). The classic symptoms of "bone, stones, abdominal groans, psychotic moans, and fatigue overtones" fits this patient's history. These symptoms, combined with an elevated serum calcium over 10.5 mg/dl, primary indicate that hyperparathyroidism is the most likely diagnosis of those listed. An elevated PTH would confirm the diagnosis. The other choices do not explain all the symptoms and the elevated calcium.

A 63-year-old female 3-day's postoperative thyroidectomy complains of tingling around her mouth and feet, and muscle spasms. You note the following finding on exam. What is the most likely cause of her symptoms and physical examination findings? Source: (McPhee and Papadakis, 2011, Chapter 26) A hypothyroidism B hypoparathyroidism C hyperkalemia D hyperparathyroidism E malnutrition

B The correct answer is (B). The patient's recent history of thyroidectomy, together with her symptoms, are suggestive of hypocalcemia due to hypoparathyroidism. Hypoparathyroidism is most commonly caused by removal of the parathyroids during thyroidectomy. The picture above is known as Trousseau sign, which, like the Chvostek Sign (facial twitching after stimulation of facial nerve) is suggestive of hypocalcemia. The other choices listed in this question would not cause Trousseau sign.

Your patient is asked to see you in followup for his fasting labs. His total cholesterol = 230 mg/dl, triglycerides = 1200 mg/dl, unable to calculate LDL. Your patient should be advised that due to these findings he is at risk for which of the following? A diabetes mellitus B pancreatitis C gout D diabetes insipidus E hypertension

B The correct answer is (B). Very elevated triglycerides, especially over 1000 mg/dl, increase the patient's risk of acute pancreatitis. This patient requires prompt treatment. Hypertriglyceridemia may be a component of metabolic syndrome but does not directly cause diabetes mellitus or hypertension. Hypertriglyceridemia does not increase the risk of gout and diabetes insipidus.

A 50-year-old male presents to your office with symptoms of weight gain and easy bruising. He is currently being treated for hypertension. No surgical history is noted and family history is noncontributory. He does not drink alcohol. Upon questioning, he also notes some difficulty with climbing stairs. He has had to use the railing more often than in the past. A photo of the patient is shown below. Which of the following blood hormones would most likely be found to be in excess? A Renin B Cortisol C Testosterone D Antidiuretic hormone E Thyroid stimulating hormone

B The correct choice is B, Cortisol. The reader must first consider Cushing's syndrome as the diagnosis for this patient. The clinical manifestations of Cushing's syndrome are noted in this patient, and include easy bruising, proximal muscle weakness, pinkish-purple wide striae, weight gain, and central obesity. These findings are related to the excess Cortisol present in the plasma of these patients. In addition, hypertension is found in 75 to 85% of patients with Cushing's syndrome

A 40-year-old female presents to the clinic with symptoms of a 10 pound unintentional weight loss, diarrhea, and palpitations. She has a positive family history for thyroid disease and diabetes mellitus. Her screening TSH level is 0.15mIU/L, and her fasting plasma glucose is 105mg/dL. What is the most common cause of this patient's symptoms? A Pituitary tumor B Graves' disease C Thyroid carcinoma D Hashimoto's thyroiditis E Euthyroid sick syndrome

B The correct choice is B, Graves' disease. Graves' disease is an autoimmune disorder, resulting in an increase in synthesis and release of thyroid hormone. It is the most common cause of hyperthyroidism

Patients prescribed which of the following medications may present with a dorsocervical fat pad, thin extremities, and central obesity, which may mimic Cushing's syndrome? A ACE inhibitors B HIV antiretrovial medications C Loop diuretics D Opioid analgesics E Aminoglycosides

B The correct choice is B, HIV antiretroviral medications. Patients using highly active antiretroviral therapy (HAART) can develop partial lipodystrophy and changes in body fat distribution. This complication occurs more commonly with long-term use, and affects 1/3 to 2/3 of patients on therapy for longer than one year. The other medication classes noted are not associated with lipodystrophy.

A 60-year-old female patient living in New York City presents for a routine office visit prior to travelling to Europe with her husband for a second honeymoon. Her medical history includes pernicious anemia, for which she is being treated with vitamin B12 supplementation. She uses no other medications. Past surgical history includes an appendectomy at age five. Upon reviewing her symptoms, you find that she has been trying to self treat for chronic constipation, without positive results. She also notes that she is easily tired and has gained 10 pounds in the last few months. On exam you note bradycardia and cool, dry skin. What is the most likely cause of her recent symptoms? A Major depression B Hashimoto's thyroiditis C Hypothalamic destruction D Toxic thyroid nodule E Iodine deficiency

B The correct choice is B, Hashimoto's thyroiditis. This autoimmune disorder is by far the most common cause of hypothyroidism in adults. This patient is presenting with classic signs and symptoms of hypothyroidism including bradycardia, cool, dry skin, and modest weight gain. She also has a history of another autoimmune disorder, pernicious anemia, which is commonly associated with hypothyroidism. Choice A, major depression, seems unlikely since the patient is planning a second honeymoon. Although many of her symptoms can be attributed to depression, her cool, dry skin is not characteristic of the disorder. Choice C, hypothalamic destruction, can cause hypothyroidism, but it is rare. Patients with choice D, toxic thyroid nodule, would typically present with symptoms of hyperthyroidism. Choice E, iodine deficiency, is a cause of hypothyroidism, but it is unlikely in patients living in developed countries with access to a variety of food sources.

A teenage girl presents to the emergency department with her parents. She has had symptoms of a urinary tract infection for the last two days, but did not tell her parents until today. She is not sexually active. Today, the girl also has diffuse abdominal pain with vomiting, general malaise, and difficulty breathing. She has no significant past medical history. Her physical exam reveals sinus tachycardia and deep fast respirations with no localization of abdominal pain or rebound tenderness. Initial lab test results reveal a plasma glucose = 378 mg/dL and serum bicarbonate = 14 mEq/L. What is the most likely diagnosis? A Hyperosmolar nonketotic hyperglycemia B Diabetic ketoacidosis C Chronic corticosteroid use D Gestational diabetes E Schmidt syndrome

B The correct choice is B, diabetic ketoacidosis (DKA). Classic signs and symptoms of this disorder include polyuria, polydipsia, marked fatigue, nausea, vomiting, signs of dehydration, fruity breath odor, postural hypotension, Kussmaul respirations, and possibly mental stupor or coma. Patients with type 1 diabetes mellitus may present for the first time in DKA. DKA is commonly precipitated by a recent infection. Choice A, hyperosmolar nonketotic hyperglycemia, is seen in patients with type 2 diabetes and presents with extremely high plasma glucose levels without acidosis and ketosis. Choice C, chronic corticosteroid use, can cause hyperglycemia and possible glucose intolerance or diabetes mellitus. This patient has no history of oral corticosteroid use. Choice D, gestational diabetes, occurs in women who are pregnant. Choice E, Schmidt syndrome, is an autoimmune polyglandular syndrome which includes diabetes mellitus in greater than 50% of patients.

A patient is admitted to the hospital for an acute adrenal crisis. He has a history of chronic adrenal insufficiency and was admitted with severe weakness, nausea and vomiting while fighting a pulmonary infection. Which of the following suggested plans might this patient have forgotten or been unable to do? A Restrict fluid intake during times of metabolic stress B Increase the daily dose of hydrocortisone during times of metabolic stress C Hold the daily dose of hydrocortisone during times of metabolic stress D Add levothyroxine to the daily dose of hydrocortisone during times of metabolic stress E Increase the daily ingestion of proteins during times of metabolic stress

B The correct choice is B, increase the daily dose of hydrocortisone during times of metabolic stress. The cortisol dose should be increased to between 60 and 80 mg/day, to mimic the normal physiologic response of the body. Increased mineralocorticoid therapy is generally not required. Choice A, restrict fluid, would aggravate the potential for the development of dehydration in this patient. Choice C would lead to further cortisol depletion. Choices D and E would not benefit this patient.

Which of the following is the most common cause of short stature worldwide? A Acromegaly B Malnutrition C Prader-Willi syndrome D Congenital growth hormone deficiency E IGF-I receptor deficiency

B The correct choice is B, malnutrition. When not associated with chronic diseases, this is the most common cause of short stature worldwide. Children with malnutrition commonly present with failure of weight gain before growth rate decreases. A dietary history is key to the diagnosis, as well as a history of any parasites in the local area. Choice A, acromegaly, is a disorder of growth hormone excess. Choice C, Prader-Willi syndrome, choice D, congenital growth hormone deficiency, and choice E, IGF-I receptor deficiency, have been found to cause short stature, but are not seen as commonly as malnutrition.

A 25-year-old woman is seen today in your office for vague abdominal pain, nausea, anorexia, weight loss, anxiety, and dizziness. Her past medical history is significant for type 1 diabetes mellitus, and her family history is significant for hypothyroidism in several family members. A review of systems reveal a history of amenorrhea. Upon exam, you note hyperpigmentation of her skin and areas of vitiligo, but no mucocutaneous candidiasis. You are not surprised to find that her serum ACTH level is elevated and her serum cotisol is low. Which of the following syndromes should be investigated in this patient? A Type 1 polyglandular autoimmune syndrome type 1 B Type 2 polyglandular autoimmune syndrome type 2 C Multiple endocrine neoplasia type 2A D Multiple endocrine neoplasia type 1 E Metabolic syndrome

B The correct choice is B, type 2 polyglandular autoimmune syndrome. This patient is presenting with signs and symptoms of adrenal insufficiency including abdominal pain, nausea, anorexia, vomiting, weight loss, anxiety, and hyperpigmentation. Type 2 polyglandular autoimmune syndrome presents most commonly in young women between 20 and 40 years old, with evidence of adrenal insufficiency, type 1 diabetes mellitus, and autoimmune thyroid disease. Because of the strong family history of thyroid disease, it would be prudent to conduct an investigation into this disorder, starting with a serum TSH level. Primary ovarian failure and vitiligo may be symptoms of the autoimmune polyglandular syndrome as well. Choice A, type 1 autoimmune polyglandular syndrome, presents more commonly in childhood with adrenal insufficiency, hypoparathyroidism, and mucocutaneous candidiasis. Choices C and D are inherited disorders, characterized by the development of several different types of endocrine organ neoplasias. Choice E, metabolic syndrome, includes a constellation of several metabolic disorders that increase the risk of cardiovascular disease and diabetes mellitus in the patient.

Your supervising physician asks you to advise him which finding is least likely to be suggestive of a thyroid malignancy in your 49-year-old female with a small palpable thyroid nodule. Which of the following choices would be least likely to suggest malignancy in this patient? A ultrasound showing lesion with microcalcifications B ultrasound showing a lesion of > 1 cm C hot nodule on 123I uptake scan D ultrasound showing a solid lesion E cold nodule on 123I uptake scan

C The correct answer is (C). A hot nodule, which is a hyperfunctioning thyroid nodule, suggests a benign etiology. The other choices, including ultrasound findings of microcalcifications, solid lesions, and lesions > 1 cm, should increase your index of suspicion for possible malignancy. Cold nodules are nonfunctioning thyroid nodules, which should increase your suspicion, especially in combination with suspicious ultrasound and/or clinical examination findings.

Your patient has a history of primary hyperparathyroidism. Recently she has been hospitalized due to obstructing kidney stones. She has had several fractures including her hip, sacrum, and forearm in the past year, all on separate occasions. She is constantly complaining of a lack of energy. What is the recommended treatment for her symptomatic hyperparathyroidism? A surgical removal of the pituitary B high-dose calcium supplementation C parathyroidectomy D thyroidectomy E thiazide diuretics

C The correct answer is (C). Primary hyperparathyroidism is most commonly secondary to a single parathyroid adenoma. The recommended treatment for symptomatic primary hyperparathyroidism is parathyroidectomy.

A patient presents with symptoms of palpitations, tremor, confusion, and sweating. She is also hungry. Her plasma glucose level is currently 54 mg/dL. Which of the following drugs would be particularly important to investigate during the patient history? A Vancomycin B Acetaminophen C Ethanol D Lisinopril E Prednisone

C The correct choice is C, ethanol. Ethanol can suppress gluconeogenesis and stimulate glucose utilization. Alcohol induced hypoglycemia occurs most commonly after a several day drinking binge with little food intake. Other medications that can be associated with hypoglycemia include insulin, insulin secretagogues, salicylates, sulfonamides, pentamidine, quinine, quinolone antibiotics, and beta 1 -adrenergic antagonists. The other choices are not associated with inducing hypoglycemia. Choice E, prednisone use, has been associated with hyperglycemia.

A 60-year-old female presents for a routine physical exam. She has no significant past medical history and is taking no medications. Her surgical history includes only carpal tunnel repair five years ago. Upon review of symptoms, she notes feeling somewhat tired. She attributes that to old age. Vital signs include BP 110/55, pulse 55 bpm, and respirations 16 per minute. Upon exam you note an enlarged, firm thyroid, thickening of her skin, and puffy facial features and pallor. A portion of blood tests that you ordered is shown below: Serum TSH = 4.9 uIU/mL Hemoglobin = 10.0 g/dL Hematocrit = 30% MCV = 101 Fasting plasma glucose = 105 mg/dL BUN = 10 mg/dL Creatinine = 0.6 ng/mL Which of the following is the most appropriate intervention? A Lithium carbonate 300 mg PO twice daily B Resection of the anterior pituitary C Levothyroxin 50 to 100 ug PO daily D Ferrous sulfate 325 mg PO three times daily E Radioactive iodine protocol

C The correct choice is C, levothyroxin 50 to 100 ug PO daily. This patient has classic signs and symptoms of hypothyroidism and required thyroid hormone supplementation. The most common form of hypothyroidism is primary hypothyroidism (e.g. Hashimoto's thyroiditis), and the most common thyroid hormone supplementation is levothyroxine.

A patient describes an insidious onset of vague gastrointestinal symptoms including nausea, vomiting, and abdominal pain. She has been losing weight over the last few weeks as well. Past medical history reveals a new diagnosis of anemia. The patient is not currently taking any medications. Physical exam findings include hypotension and dark freckles with dark, bluish black pigmentation of the mucosal membranes. Serum sodium is decreased, while the potassium and plasma ACTH levels are elevated. What is the most likely diagnosis for this patient? A Acute abdomen B Cushing's syndrome C Primary adrenal insufficiency D Disseminated intravascular coagulation E Diabetic ketoacidosis

C The correct choice is C, primary adrenal insufficiency. This disorder presents with symptoms resulting from progressive destruction of the adrenal glands and resultant decrease in aldosterone and cortisol secretion. The presentation can range from subtle to fulminant adrenal crisis. Common clinical findings include hyperpigmentation, weakness, anorexia, nausea, vomiting, diarrhea, and hypotension. Some patients with adrenal insufficiency may initially be treated for an acute abdomen, choice A. In this patient, ACTH levels are known to be elevated, pointing the cause to the pituitary-adrenal axis. Choice B, Cushing's syndrome, presents with signs and symptoms related to an excess in plasma cortisol, such as truncal obesity, hyperglycemia, hypertension, skin changes, and weight gain. Choice D, disseminated intravascular coagulation, is a coagulation disorder presenting with bleeding and thrombosis. Choice E, diabetic ketoacidosis, is an acute complication seen in patients with type 1 diabetes mellitus.

A 60-year-old female presents to her primary care practitioner for her yearly check up. Her past medical history is significant only for a history of repeat kidney stones. She enjoys gardening outside, has no significant family history, and is not taking any medications. Routine mammograms have been normal. During the review of her systems, the patient describes feeling tired lately and has noticed muscle aches over the last few weeks. Initial lab values are shown below: Hemoglobin = 12.0 gm/dL Hematocrit = 36% BUN = 10 mg/dL Creatinine = 0.7 ng/mL Calcium = 12 mg/dL Phosphate = 2.0 mg/dL PTH = 100 pg/mL TSH = 2.0 UIU/mL What is the most likely cause of her symptoms? A Vitamin D deficiency B Hypercalcemia of malignancy C Primary hyperparathyroidism D Factitious hypercalcemia E Hyperthyroidism

C The correct choice is C, primary hyperparathyroidism. This patient has the characteristic signs and symptoms of hypercalcemia, along with the lab findings of primary hyperparathyroidism with elevated calcium, low phosphate, and elevated PTH (normal values being 10-65 pg/ml). Other common presenting features include bone pain, paresthesias, polyuria, and gastrointestinal symptoms. Females are affected three times as much as males, and it is typically found accidentally.

What is the definitive treatment for the majority of patients presenting with mild symptoms of hyperthyroidism secondary to subacute thyroiditis? A Subtotal thyroidectomy B Oral methimazole C Symptomatic treatment D Radioactive iodine E Antibiotics

C The correct choice is C, symptomatic treatment. Most patients with subacute thyroiditis and symptoms of hyperthyroidism require only symptomatic treatment, with non-steroidal anti-inflammatory medications and/or beta blockers, for any cardiac symptoms including palpitations and tachycardia. Occasionally, patients may require a course of prednisone for this acute inflammatory condition. Most patients will recover spontaneously within a few months. Choices A, B, D, and E are not necessary in this condition. Most cases of subacute thyroiditis are associated with viral infections, and resolve without additional thyroid medications.

A patient presents with symptoms of polyuria for several months. There is no history of diabetes mellitus or intrinsic kidney disease. Which of the following tests can best determine whether the polyuria is resulting from primary polydipsia or some form of diabetes insipidus? A 24-hour urine volume test B Routine urinalysis C Water deprivation test D Urine culture and sensitivity E Vasopression suppression test

C The correct choice is C, water deprivation test. This test helps the practitioner determine if the patient can concentrate his/her urine with or without becoming hyperosmolar. The patient is restricted from liquids and food during the test, which must be done in a controlled environment. A patient with primary polydipsia will be able to concentrate his/her urine without becoming hyperosmolar. A patient with diabetes insipidus will become hyperosmolar without concentrating the urine.

An 88-year-old female patient has been advised by her primary care physican that she needs a computed tomography (CT) scan of her abdomen and pelvis due to persistent abdominal pain, bloating, and weight loss. She was told that she needs to hold one of her medications the day of the procedure and that she may resume the medication 48 hours later. She can't remember which medication she should discontinue. Which medication listed below should she discontinue temporarily as specified above due to the diagnostic test ordered? A glyburide B glipizide C pioglytizone D metformin E acarbose

D The correct answer is (D). A CT of the abdomen and pelvis requires p.o. and IV iodinated contrast unless ordered specifically without IV contrast. There is an increased risk of acute renal failure with IV iodinated contrast. The risk to the patient may increase with metformin and therefore should ideally be held prior to and for 48 hours after any radiocontrast IV study to avoid the added possibility of lactic acidosis

A 14-year-old boy presents to the emergency department with his parents. He has a history of type 1 diabetes, and has had bronchitis for the last few days. He is now presenting with difficulty breathing, worsening fatigue, polydipsia, and polyuria. His last fingerstick glucose at home this morning was 350 mg/dL. Which of the following patterns of breathing are characteristic of this complication of diabetes? A Cheyne stokes respiration B Bradypnea C Biot breathing D Kussmaul breathing E Painful respiration

D The correct choice is D, Kussmaul breathing, which is deep regular breathing or hyperpnea. It can be seen as a compensatory action of metabolic acidosis and hypoxia. Choice A, Cheyne-Stokes respiration, is a waxing and waning pattern of rate and volume that includes periods of apnea. This can be seen in patients at high altitudes, and with severe left sided heart failure or neurologic disease. Choice B, bradypnea, is noted with a slower than usual respiratory rate and can be seen with use of CNS depressant drugs, uremia, or structural intracranial lesion. Choice C, Biot breathing, is an uncommon variant of Cheyne-Stokes respiration, with periods of apnea alternating with a series of equal breaths that end abruptly. It can be seen in patients with meningitis. Choice E, painful respiration, is relatively normal in pattern, but interrupted by pain during breathing from such disorders as pleurisy, fractured ribs, or subphrenic inflammation.

A 30-year-old female presents to your office for a routine physical exam. She has not seen a health care provider in many years. Upon talking with the patient, you find out that she had been diagnosed with hypertension several years ago, but was unable to afford the antihypertensive medications that were prescribed to her. She has no complaints at this time. Upon exam of the head and neck, you note widened spaces between her lower incisor teeth and a large, fleshy nose. Her skin is oily and she demonstrates mild proximal muscle weakness. Her EKG reveals a left axis deviation and widened QRS. What is the most likely rationale for her clinical presentation? A Diabetes mellitus B Cushing's syndrome C Hypothyroidism D Acromegaly E Clinical depression

D The correct choice is D, acromegaly. Patients with acromegaly have an abundance of growth hormone secretion. This leads to excessive growth of many areas of the body including soft tissue. Patients with acromegaly also have an increased incidence of hypertension and left ventricular hypertrophy. None of the other choices will cause this patient's constellation of symptoms. Patients with many endocrine disorders may develop weaknesses as seen in this patient, but the large nose and widely spaced teeth are characteristic of acromegaly.

Which of the following is the most common cause of primary adrenal insufficiency in the United States? A Tuberculosis B Adrenal hemorrhage C Lymphoma D Autoimmune destruction E Metastatic carcinoma

D The correct choice is D, autoimmune destruction. This is responsible for 80% of cases of primary adrenal insufficiency in the United States. All of the other choices can cause adrenal insufficiency, but they are less common. Tuberculosis, choice A, is a common cause of adrenal insufficiency in other areas of the world, where the infection is more common. Bilateral adrenal hemorrhage, choice B, can occur as a complication of sepsis, heparin use, anti-phospholipid syndrome, and after major trauma or surgery. Lymphoma, choice C, and metastatic carcinoma, choice E, are rare causes of adrenal insufficiency.

To which of the following areas does follicular thyroid cancer most commonly first spread? A Intraglandular metastasis B Local spread into regional vocal cords C Distant lymph nodes D Bone and lung via bloodstream E Local extension into the muscle and trachea

D The correct choice is D, bone and lung via bloodstream. Follicular thyroid cancer can spread to regional lymph nodes and distant sites via the blood stream. Choice A, intraglandular metastasis, is seen more commonly in papillary thyroid carcinoma, and choice B, local spread into regional vocal cords, occurs in anaplastic thyroid carcinoma. Choice C, distant lymph nodes, and choice E, local extension into the muscle and trachea, are more commonly seen in patients with medullary thyroid cancer.

A young man presents with difficulty breathing at times. Upon exam you note evidence of a firm, fixed thyroid nodule with extension toward the trachea and surrounding muscles. The patient has a family history of thyroid cancer. You are concerned that the patient may have medullary thyroid cancer. After you confirm your suspicion, which of the following genetic tests would you suggest to his family members? A BRCA 1 B APC C CFTR D ret proto-oncogene E CAG triplet expansion

D The correct choice is D, ret proto-oncogene. Genetic testing to identify this oncogene is available for patients whom you suspect have medullary thyroid cancer and/or pheochromocytoma, and for their asymptomatic families who may be at risk for the same disorders. A mutation in this gene leads to uncontrolled growth of the C cells of the thyroid. Choice A, BRCA 1, is an oncogene that is associated with risk for breast, ovarian, colon, and prostate cancers. Choice B, APC, is an oncogene noted in the familial colon cancer syndrome known as familial adenomatous polyposis. DNA analysis for CFTR mutations, choice C, can identify the gene mutations in patients with cystic fibrosis in the vast majority of instances. Choice E, CAG triplet expansion, is the result of a gene mutation in Huntington's disease.

What is the most sensitive test available for the screening and detection of early thyroid dysfunction? A Radioactive iodine uptake B Serum T3 resin uptake C Serum total T4 level D Serum TSH level E Thyroid scan

D The correct choice is D, serum TSH level. Very small changes in serum TSH level can provide clues that there are changes in the functioning of the hypothalamic-anterior pituitary-thyroid axis. The test is easier and less expensive than any thyroid imaging tests. All of the other choices can be used in the work up of patients for thyroid dysfunction, but they are less sensitive, and many are more expensive and more invasive.

A female patient presents with a history of hypertension and low plasma HDL. She is asking you if she has metabolic syndrome. She does not have diabetes and she is not obese. Which of the following NCEP ATP III criteria would be needed to confirm that diagnosis? A LDL > 70 mg/dL B LDL > 90 mg/dL C Triglycerides > 100 mg/dL D Triglycerides ≥ 150 mg/dL E Triglycerides > 300 mg/dL

D The correct choice is D, triglycerides ≥ 150 mg/dL. The NCEP ATP III criteria for metabolic syndrome include three or more of the following: Central obesity with waist circumference > 102 cm in men and > 88 cm in women. Hypertriglyceridemia with serum triglycerides ≥ 150 mg/dL, or the patient is taking medication for hypertriglyceridemia. Low HDL level < 40 mg/dL in men and < 50 mg/dL in women, or the patient is taking medication for low HDL Hypertension with blood pressure ≥ 130 mm systolic or ≥ 85 mm diastolic, or the patient is taking medication for high blood pressure. Fasting plasma glucose ≥ 100 mg/dL, or a positive diagnosis for diabetes mellitus, or the patient is taking medication for hyperglycemia.

What is the radionuclide imaging pattern noted during a thyroid scan in patients with subacute thyroiditis? A Single area of increased uptake B Diffusely high uptake C Multiple areas of increased uptake D Single area of low uptake E Diffusely low uptake

E The correct choice is E, diffusely low uptake. Acute inflammation is occurring during subacute thyroiditis, causing leakage of stored thyroid hormone into the circulation. The thyroid is not metabolically active, and therefore there is low radioactive iodine uptake on scan. This is in distinction to the diffuse increased uptake seen on scan in Graves' disease, choice B. Choice A, a single area of increased uptake, is commonly seen with a "hot" or toxic nodule. Choice C, multiple areas of increased uptake, are found in patients with multinodular goiters. This patient had no nodules noted on exam. Choice D, single area of low uptake, is seen as cold nodules and may be related to a benign nodule or thyroid cancer.

A 47-year-old man presents with signs and symptoms of Cushing's syndrome. He has no history of depression, alcoholism, anorexia, or medication use. His past medical history is negative for any known malignancies. An MRI of the pituitary gland is not definitive for any masses. You are trying to decide if the symptoms are caused by an occult pituitary adenoma or occult malignancy in another area of the body. Which of the following is the definitive test used to differentiate pituitary from non-pituitary ACTH dependent Cushing's syndrome? A 12:00 a.m. plasma cortisol B Random plasma ACTH C 24-hour urine free cortisol D MRI of the adrenal glands E Inferior petrosal sinus ACTH

E The correct choice is E, inferior petrosal sinus ACTH. This test can definitively indicate whether or not an elevated plasma ACTH level is due to secretion from the pituitary or an ectopic source. Simultaneous measurements of ACTH from the peripheral circulation and the inferior petrosal sinus after CRH stimulation can help to locate the surge's origin. Highly accurate results can be obtained if this test is performed by a skilled interventional radiologist. Although the other choices listed are used in the evaluation of patients with hypercortisolemia, they do not detect any related ACTH surge source.

A patient is recovering from having a total thyroidectomy two days ago for medullary thyroid cancer. An extensive neck dissection was required during the surgery. Post-operative lab testing reveals a low serum calcium level. Which of the following clinical presentations will most likely occur in this patient? A Constipation B Anorexia C Polyuria D Bone pain E Paresthesias

E The correct choice is E, paresthesias. Circumoral paresthesias are signs of hypocalcemia. Hypocalcemia can occur after any type of neck surgery that may have resulted in destruction of the parathyroid glands. Choices A through D are symptoms of hypercalcemia and may be seen in hyperparathyroidism.

What is the definitive treatment of choice for elderly patients diagnosed with Graves' disease? A Beta blocking agents B Levothyroxine C Methimazole D Total thyroidectomy E Radioactive iodine

E The correct choice is E, radioactive iodine. This is the treatment of choice in the elderly because it is efficient, easy to take, and inexpensive. Choice A, beta blocking agents, are useful in the treatment of symptoms of hyperthyroidism, such as palpitations, but they are not a definitive treatment for the disorder. Choice B, levothyroxine, is used for thyroid hormone supplementation in patients with hypothyroidism. Choice C, methimazole, is an anti-thyroid drug that has increased toxicity in the elderly and is more useful in younger patients with mild hyperthyroidism. Choice D, total thyroidectomy, has a limited role as a treatment for hyperthyroidism, and is associated with increased morbidity in the elderly.

A 40-year-old patient presents to your walk-in clinic with symptoms of hyperhydrosis, oily skin, daytime sleepiness, and snoring. Upon exam, you note large fleshy heel pads and hands with sweaty palms. The patient also has coarse facial features. When asked, the patient isn't aware of any major changes in her face or body. She has not seen another health care provider in many years and has not kept up with any health care maintenance schedule. The patient lives with her ill mother and is not currently employed. Which of the following screening tests would best aid in the diagnosis of this patient? A CT of the chest and abdomen B Thyroid scan C 24-hour urine for catecholamines D Serum calcitonin level E Serum IGF-I level

E The correct choice is E, serum IGF-I level. Age and gender matched levels of IGF-I are elevated in patients with acromegaly. IGF-I is the mediator of most of the effects of GH on the body, and lead to the proliferation of bone, cartilage, and soft tissue. Although GH levels may be elevated in patients with acromegaly, they are secreted in a pulsatile fashion and are not consistently elevated. Serum GH levels are not the best screening test for acromegaly.

A 40-year-old patient presents to your walk-in clinic with symptoms of hyperhydrosis, oily skin, daytime sleepiness, and snoring. Upon exam, you note large fleshy heel pads and hands with sweaty palms. The patient also has coarse facial features. When asked, the patient isn't aware of any major changes in her face or body. She has not seen another health care provider in many years and has not kept up with any health care maintenance schedule. The patient lives with her ill mother and is not currently employed. Which of the following screening tests would best aid in the diagnosis of this patient? A CT of the chest and abdomen B Thyroid scan C 24-hour urine for catecholamines D Serum calcitonin level E Serum IGF-I level

E The correct choice is E, serum IGF-I level. Age and gender matched levels of IGF-I are elevated in patients with acromegaly. IGF-I is the mediator of most of the effects of GH on the body, and lead to the proliferation of bone, cartilage, and soft tissue. Although GH levels may be elevated in patients with acromegaly, they are secreted in a pulsatile fashion and are not consistently elevated. Serum GH levels are not the best screening test for acromegaly. Choices A, a CT of the chest and abdomen, and B, a thyroid scan, are expensive imaging studies that are not usually used as screening tests. They also have no role in the routine workup of patients with suspected acromegaly. Choice C, 24-hour urine for catecholamines, is a test that can be used in the work up of patients with suspected pheochromocytoma. Choice D, serum calcitonin levels, are associated with medullary thyroid cancer and other thyroid disease.

A patient presents to the emergency department with signs and symptoms of acute pancreatitis. He does not drink alcohol and has no history of gallstones. He has a positive history of increased lipids. Very high levels of which of the following lipids is associated with an increased risk of pancreatitis? A High density lipoprotein B Low density lipoprotein C Apolipoprotein C D Total cholesterol E Total triglycerides

E The correct choice is E, total triglycerides. Very high levels of triglycerides increase the risk of acute pancreatitis, probably from local release of free fatty acids and lysolecithin from lipoprotein substrates in the pancreatic bloodstream. If the albumin binding capacity is exhausted, these fatty acids can initiate a chemical pancreatitis and destroy parenchymal cells in the area.

A 10-year-old child is seen with his parents for a routine check up. During the review of symptoms, his parents mention that their son has been extremely thirsty and is going to the bathroom to urinate frequently. The patient agrees. The parents are concerned that their son has developed diabetes mellitus. The family history is negative for diabetes mellitus, but the mother has a history of familial hypothalamic diabetes insipidus. Screening blood work includes a CBC, hemoglobin A1c, and renal function tests, all of which are within the reference range. Which of the following serum analytes would you expect to be deficient? A Sodium B Glucose C Thyroxine D Prolactin E Vasopressin

E. Vasopressin

26-year-old woman has decreased appetite, weight gain, cold intolerance, hoarse voice, constipation, and arthralgias. What is the most likely etiology of her condition? A autoimmune thyroiditis B congenital hypothyroidism C dietary iodine deficiency D surgical resection of the thyroid gland

The Correct Answer is: A Auto immune thyroiditis is the most common cause of hypothyroidism in the United States. Dietary iodine deficiency is the most common cause in underdeveloped regions of the world

Which of the following drugs can cause syndrome of inappropriate antidiuretic hormone (SIADH)? A carbamazepine B glyburide C lithium carbonate D metoprolol

The Correct Answer is: A Many medications can enhance the release or potentiate the effects of ADH. Carbamazepine may increase ADH release.

A 62-year-old obese woman presents with progressive numbness and tingling in her feet for the past 3 months. On physical examination, the patient is found to have decreased sensation to pinprick and vibration, absence of ankle reflexes, and difficulty with tandem walking. Which is the most common etiology of her symptoms? A diabetes mellitus B alcoholism C vitamin B12 deficiency D spinal cord tumor E rheumatoid arthritis

The Correct Answer is: A Peripheral neuropathy is a syndrome that is manifested by muscle weakness, paresthesias, decreased deep tendon reflexes, and autonomic disturbances most commonly in the hands and feet, such as coldness and sweating. There are many causes of peripheral neuropathy ranging from metabolic conditions to malignant neoplasm, rheumatoid arthritis, and drug and alcohol use. The increase in non-insulin-dependent diabetes mellitus due to obesity in the American population has increased the incidence of associated disease states.

A 76-year-old man, is brought to the emergency department by his niece after she found him wandering around his yard in the cold wearing only a tee shirt and jeans. When she set up his pill container about 36 hours earlier, he seemed his usual self but, in retrospect, possibly a little more confused than usual. The niece says that he has "high blood," treated with a "white fluid pill," "sugar diabetes," treated with an oral medication, and early "old timer's" dementia treated with "a memory pill." Vital signs include an oral temperature of 100.8F, pulse 100 beats per minute, respirations 24 and somewhat shallow, and blood pressure of 88/52. Initial examination reveals a slightly dehydrated, stuporous man appearing older than his stated age, who smells strongly of urine. He has no lateralizing signs. What is the most likely cause of the mental status changes? A hyperglycemic hyperosmolar state B lactic acidosis C stroke D urinary tract infection E worsening dementia

The Correct Answer is: A The combination of confusion and dehydration in a patient with diabetes type 2 who is taking a diuretic strongly suggest hyperosmolar state. Patients with lactic acidosis (B) have marked hyperventilation and, usually, signs and symptoms of a serious illness. The lack of lateralizing signs makes a stroke (C) less likely. Urinary tract infection (D) could certainly cause confusion and incontinence in an elderly man and should be investigated. Alzheimer dementia (E) progresses slowly; sudden decompensation is usually due to delirium

A 28-year-old woman presents with nervousness and palpitations associated with heat intolerance. On examination, there is no evidence of thyromegaly, but there is a palpable nodule that is "hot" on a thyroid scan. The TSH was low and T3 and T4 were both elevated. Which of the following is the recommended treatment for this patient? A Propylthiouracil (PTU) B Thyroid lobectomy C Total thyroidectomy D Radiation therapy

The Correct Answer is: B In Graves' disease, the thyroid is diffusely enlarged in contrast to a toxic adenoma in which the thyroid is normal sized but with a palpable nodule. Surgery is the treatment of choice for a toxic adenoma. Surgical treatment of a toxic adenoma is a thyroid lobectomy and isthmusectomy. A subtotal or total thyroidectomy is indicated for toxic multinodular goiters or Plummer disease. Thionamides and radioiodine ablation are not effective therapies for toxic adenomas.

A 29-year-old woman comes in for evaluation of "panic attacks." She has no history of anxiety or depression. She says that during these episodes, which have been getting more frequent and more severe over the past month, she perspires heavily, feels highly anxious ("as though the end is coming"), and as if her heart is "going to jump out of my chest." Recently she has also begun experiencing headaches during these attacks. Her best friend told her that her face gets really "blotchy" during the attacks and then "awfully red" afterwards. Her examination today is remarkable only for blood pressure of 160/100. What is the most sensitive test for diagnosing her condition? A plasma fractionated free metanephrines B serum chromogranin A C serum thyroid stimulating hormone D urine fractionated metanephrine and creatinine E urine toxicology screen

The Correct Answer is: A The plasma fractionated free metanephrine test is the most sensitive test for a pheochromocytoma which this woman's symptoms strongly suggest. Serum chromogranin A (B) is elevated in about 90% of patients and its level correlates with tumor size. Serum TSH (C) would not be appropriate since the symptoms are not as suggestive of hyperthyroidism as they are of pheochromocytoma. A positive plasma test should be followed by a urine fractionated metanephrine test and creatinine level (D). A urine toxicology screen (E) would be appropriate if cocaine use were suspected; however the symptoms more strongly suggest pheochromocytoma.

A 30-year-old man is brought to the emergency department by his wife for abdominal pain, nausea, vomiting, and diarrhea. She says he has been getting "worse and worse" for at least the past two months. He is "too tired" and his "muscles hurt too much" to play golf, formerly their favorite leisure activity. He has been depressed and reticent, instead of his usual talkative self. He won't eat much of anything, even when she cooks his favorite meals and she is sure he has lost considerable weight because his clothes "hang off him." He has refused medical evaluation until today, when the abdominal pain of approximately a week increased and he began having the vomiting and diarrhea. Initial impression is that the man is quite tanned, although it is the middle of winter. Closer examination reveals dark pigmentation in his skin folds and on the buccal mucosa. This presentation most strongly suggests which of the following? A adrenocortical insufficiency B diabetic ketoacidosis C hypercortisolism D hyperglycemic hyperosmolar state E hypothyroidism

The Correct Answer is: A This is a fairly classic presentation of adrenocortical insuffiency (Addison's disease).

A 29-year-old woman comes to the office because she "just keeps gaining weight and can't stop." Since she was last seen in the office at age 24, she has gained nearly 100#. Physical examination is remarkable for blood pressure of 140/92. She appears depressed. Her trunk and abdomen are heavy with normal-sized extremities. Her facial hair is dark and in a "male" distribution. Large dark violaceous striae are present on her abdomen and proximal extremities. Which of the following additional findings is most consistent with this presentation? A Buffalo hump B Doughy, thickened skin C Exophthalmos D Lid lag E Thickened tongue

The Correct Answer is: A This woman appears to have Cushing Syndrome (hypercortisolism) which is characterized, in addition to the signs listed above, by plethoric facies, supraclavicular fatpads, and the so-called "buffalo hump." Doughy, thickened skin (B) and thickened tongue (E) may be found in hypothyroidism, exophthalmos (C) in Graves disease, and lid lag (D) in hyperthyroidism from any cause.

A 75-year-old man with type 2 diabetes presents to the emergency department with a 2-day history of confusion and lethargy. On physical exam, notable dehydration, tachycardia, and confused mental state is noted. Serum sodium, potassium, magnesium, and chloride levels are normal. The arterial blood gases are normal and serum ketones are negative. The abnormal laboratory findings are as follows: Given this information, what is the most likely diagnosis? A diabetic ketoacidosis B hyperglycemic hyperosmolar state C hypoglycemia D dehydration

The Correct Answer is: B A hyperglycemic hyperosmolar state is characterized by dehydration, significant hyperglycemia, and an elevated serum osmolality with an insignificant or negative ketosis. Because of the lack of ketosis, the patient may present with a gradual onset of symptoms, and it can go unnoticed until the dehydration becomes more severe than in ketoacidosis.

In a patient who does not have thyroid disease, an elevated serum thyroid stimulating hormone (TSH) may be found under which of the following conditions? A acute corticosteroid administration B acute psychiatric admission C development of an hCG-secreting tumor D pregnancy E use of amphetamines

The Correct Answer is: B About 15% of patients who are admitted for an acute psychiatric illness will have an elevated TSH in the absence of thyroid disease. Acute corticosteroid administration (A), hCG-secreting tumors (C), pregnancy (D), and use of amphetamines (E) are all associated with low TSH levels.

You suspect that a 44-year-old female with a history of fatigue, anxiety, and weight loss has hyperthyroidism. What physical examination findings would support hyperthyroidism as your diagnosis? A dry skin B exophthalmos C decreased tendon reflexes D thinning hair E bradycardia

The Correct Answer is: B Exophthalmos is a physical examination finding that is found in some patients with hyperthyroidism due to Grave's disease. Other potential physical examination findings may be a tremor, moist skin, and tachycardia. The other choices may be found in patients with hypothyroidism.

A 28-year-old woman presents with nervousness and palpitations associated with heat intolerance. On examination, there is no evidence of thyromegaly, but there is a palpable nodule that is "hot" on a thyroid scan. The TSH was low and T3 and T4 were both elevated. Which of the following is the recommended treatment for this patient? A Propylthiouracil (PTU) B Thyroid lobectomy C Total thyroidectomy D Radiation therapy

The Correct Answer is: B In Graves' disease, the thyroid is diffusely enlarged in contrast to a toxic adenoma in which the thyroid is normal sized but with a palpable nodule. Surgery is the treatment of choice for a toxic adenoma. Surgical treatment of a toxic adenoma is a thyroid lobectomy and isthmusectomy. A subtotal or total thyroidectomy is indicated for toxic multinodular goiters or Plummer disease.

In addition to insulin and fluid replacement with 0.9% saline, which electrolyte is commonly infused in the type 2 diabetic patient who arrives in the emergency department in a hyperglycemic, hyperosmolar, nonketotic state? A bicarbonate B potassium C calcium D magnesium E sulfate

The Correct Answer is: B Insulin not only causes cellular uptake of glucose but also of potassium. Hypokalemia may develop when insulin is infused to correct either a hyperglycemic hyperosmolar state or a diabetic ketoacidosis. Hence, in order to avoid hypokalemia, potassium chloride can be added to a saline solution, as long as the serum potassium is not elevated

A 41-year-old woman presents with complaints of weight gain, infrequent menses, and mood changes. You observe her to have moon facies, centripetal fat distribution, and purple striae on her abdomen (see Figure 4-3). Her blood pressure is 152/98 mm Hg. What is the first step in confirming this diagnosis? (Reproduced, with permission, from Fauci AS, Braunwald E, Kasper DL, et al. Harrison's Principles of Internal Medicine, 17th edition. New York: McGraw-Hill, 2008: 2255.) A random cortisol level B overnight dexamethasone suppression test C thyroid studies D MRI

The Correct Answer is: B This patient's signs and symptoms indicate possible Cushing syndrome. Overnight dexamethasone testing is the most widely used test, with normal results excluding Cushing syndrome. Cortisol levels are not useful because of diurnal variations

A 45-year-old male comes into your family practice office for his second follow-up appointment since being diagnosed with type 2 diabetes. He does not complain of any symptoms. He is currently taking metformin 1000 mg BID and his HgA 1c at this visit 7.6%. You ask him about his diet and if he is regularly monitoring his blood glucose. He says that he does not really watch what he eats, but he does check his glucose levels daily. He tells you that on average his fasting plasma glucose (FPG) is usually around 88 mg/dl and his postprandial glucose (PPG) is around 180 mg/dl. What would be the BEST next appropriate step in management for this patient? A Nothing, his levels are within normal limits. B Consider switching to basal insulin therapy to control his FPG. C Add a dipeptidyl peptidase 4 (DPP-4) inhibitor to control his PPG. D Add a sulfonylurea to control his PPG. E Increase his metformin to 1500 mg twice daily.

The Correct Answer is: C DPP-4 inhibitors modulated glucagon-like peptide-1 (GLP-1). Their mechanism of action is thought to result from increased incretin levels, especially GLP-1. GLP-1 inhibits glucagon release which, in turn, results in increased insulin secretion, delays gastric emptying, and decreases serum glucose levels. The class is particularly appropriate to utilize in patients who have near-normal HbA 1C and elevated postprandial serum glucose because they work only when food enters the gut and have little to any chance of hypoglycemia. Sulfonylurea is more likely to result in hypoglycemia

A 30-year-old patient presents 2 months postthyroidectomy. The patient has had symptoms of increased irritability, muscle spasms, and hair loss for the past month. On physical examination, a positive Chovstek sign is noted. Which of the following is the most likely diagnosis? A hypothyroidism B hypopituitarism C hypoparathyroidism D hypogonadism

The Correct Answer is: C Hypoparathyroidism commonly presents following thyroidectomy surgery. This patient has classic signs and symptoms of a low calcium level and hypoparathyroidism. Chovestek sign is a physical exam finding that is positive after tapping in front of the ear in the facial nerve region. When doing this, the muscle contracts. When the calcium level is low, this occurs. Hypothyroidism can occur following a thyroidectomy but the symptoms are not the same

A 70-year-old woman who was found barely responsive at home by her daughter is brought to the emergency department. Evaluation reveals that she is in a hyperglycemic hyperosmolar state with a severe fluid deficit. Treatment is initiated with vigorous saline rehydration and a continuous infusion of insulin. At what point should her glucose be added to her treatment? A when her condition becomes stable B when her urine output reaches 50 mL/hour C when her blood glucose reaches 250 mg/dL D if she develops hypokalemia E if she begins to spill ketones in her urine

The Correct Answer is: C In hyperglycemic hyperosmolar states, the serum glucose rapidly corrects with fluid administration alone. However, with vigorous rehydration, glucose may fall precipitously and lead to severe hypoglycemia. To avoid this, glucose should be added to water, half-normal, or normal saline as soon as the patient's blood glucose is less than or equal to 250mg/dL. She should continue to receive insulin IV until she is stabilized (A) when it can be switched to subcutaneous administration. The goal of fluid therapy in this patient is restoring her urine output to 50 mL per hour (B) or more. Because insulin drives potassium into the cells and can cause hypokalemia (D), potassium chloride should be given unless the patient has chronic kidney disease or oliguria. Persons in a hyperglycemic hyperosmolar state typically do not spill ketones (E) the way persons with diabetic ketoacidosis do.

A 43-year-old obese man presents for a health maintenance visit. On physical exam, it is noted that his waist circumference is 106 cm and blood pressure is 148/92 mm Hg. Which of the following fasting laboratory levels would suggest a diagnosis of metabolic syndrome (syndrome X) in this patient? A HDL of 45 mg/dL B LDL of 180 mg/dL C triglyceride of 190 mg/dL D glucose of 100 mg/dL

The Correct Answer is: C Metabolic syndrome is found in approximately 25% of Americans. It is defined as three or more of the following findings: waist circumference of greater than 102 cm in men or greater than 88 cm in women; serum triglyceride level of at least 150 mg/dL, HDL level of less than 40 mg/dL in men or less than 50 mg/dL in women; blood pressure of at least 130/85 mm Hg; and serum glucose level of at least 110 mg/dL.

A 43-year-old pilot is interested in quitting his 20-year habit of smoking. His medical history includes type 2 diabetes mellitus diagnosed 6 years ago for which he is currently taking metformin. Which of the following would be most appropriate to recommend to this patient? A alprazolam B clonidine C nicotine replacement therapy D nortriptyline E varenicline

The Correct Answer is: C Nicotine replacement therapy is relatively safe in the majority of patients and comes in many forms (transdermal patches, gums, sprays, and inhalers). Both clonidine and nortriptyline are considered second-line smoking cessation agents because of their many side effects. Neither has been approved by the FDA for smoking cessation. Alprazolam is also not indicated, and there is currently no evidence that it aids in smoking cessation. Varenicline is a relatively new agent for smoking cessation and is a partial agonist to α 4 -β 2 nicotinic acetylcholine receptors. It has been approved by the FDA; however, varenicline is banned from use by pilots and air traffic controllers as per the Federal Aviation Administration (FAA

A 45-year-old man with a history of neck irradiation for Hodgkin lymphoma at the age of 15 is found to have a 1.5-cm, nontender, firm thyroid nodule. Upon laboratory evaluation, the patient is found to be euthyroid, and fine needle biopsy reveals malignancy. What histologic type is most likely? A anaplastic B follicular C medullary D papillary

The Correct Answer is: D Thyroid carcinoma often presents as an asymptomatic thyroid nodule. The most common histologic form is papillary carcinoma, representing more than 80% of cases.

A 43-year-old pilot is interested in quitting his 20-year habit of smoking. His medical history includes type 2 diabetes mellitus diagnosed 6 years ago for which he is currently taking metformin. Which of the following would be most appropriate to recommend to this patient? A alprazolam B clonidine C nicotine replacement therapy D nortriptyline E varenicline

The Correct Answer is: C Nicotine replacement therapy is relatively safe in the majority of patients and comes in many forms (transdermal patches, gums, sprays, and inhalers). Both clonidine and nortriptyline are considered second-line smoking cessation agents because of their many side effects. Neither has been approved by the FDA for smoking cessation. Alprazolam is also not indicated, and there is currently no evidence that it aids in smoking cessation. Varenicline is a relatively new agent for smoking cessation and is a partial agonist to α 4 -β 2 nicotinic acetylcholine receptors. It has been approved by the FDA; however, varenicline is banned from use by pilots and air traffic controllers as per the Federal Aviation Administration (FAA)

Which of the following medications for type 2 diabetes mellitus is thought to best preserve beta cell function in the pancreas? A acarbose B glimepiride C pioglitazone D sitagliptin E insulin glargine

The Correct Answer is: C Pioglitazone is a thiazolidinedione (TZD). TZDs are thought to help preserve beta-cell function.

A 23-year-old patient with type 1 diabetes mellitus (DM) has been having difficulty sleeping at night. Usually around 3 am the patient will wake up feeling sweaty, nauseated, and tachycardic. He has recorded the following blood glucose levels: What advise is the best for this patient? A stop eating a bedtime snack B increase the evening regular dosage C decrease the evening Lente dosage D exercise before going to bed at night

The Correct Answer is: C The patient has described the Somogyi effect. This effect occurs because the patient is receiving too much intermediate insulin at dinnertime. This occurs when nocturnal hypoglycemia results in counter-regulatory hormones producing hyperglycemia. Either the intermediate insulin dosage can be shifted to a lower dosage at bedtime or the patient can eat a larger snack at bedtime.

A 36-year-old female patient is being treated for thyroid disease and returns to your family practice office for a six month exam. She is on 100 ugm of levothyroxine once daily. Her vital signs are a blood pressure of 128/82 mm&thinsp;Hg, pulse 62 bpm, respirations 12 bpm, and she is afebrile. Her physical examination is normal. Her thyroid-stimulating hormone (TSH) level is 11.5 uIU/mL (normal 0.4 to 5.0 uIU/mL). What is your next step in her treatment? A Draw a free T3 and free T4. B Lower her levothyroxine to 88 ugm daily and recheck her TSH in 4 to 6 weeks to adjust if necessary. C Increase her levothyroxine to 112 ugm daily and recheck her TSH in 4 to 6 weeks to adjust if necessary. D She is asymptomatic so do not adjust her levothyroxine, but recheck her level at her next visit in six months.

The Correct Answer is: C The thyroid gland operates on a reverse feedback mechanism. When TSH is high, free T 3 /T 4 is low and need repletion, and vice versa. In this case, the patient has an elevated TSH, indicating that her serum T 3 /T 4 is likely low and she should have her dose of levothyroxine adjusted upwards and levels can be rechecked starting in approximately four weeks and adjusted subsequently as necessary.

A 55-year-old man patient presents with tachycardia and heart palpitations. Physical exam shows a multinodular goiter. He does not have obstructive symptoms. He has suppressed TSH and elevated T 3 and T 4 , and a thyroid scan shows multiple functioning nodules. What is the treatment of choice for this patient? A propylthiouracil B beta-blockers C 131I ablation D surgical resection

The Correct Answer is: C The treatment of choice for multinodular goiter is 131 I ablation. In patients with very large thyroid glands with obstructive symptoms, surgical resection may be the best option.

A 23-year-old woman, adopted in infancy, has just learned that her biologic family has a long history of tumors of the thyroid and parathyroid glands, the pancreas, the adrenal glands, and of lipomas. Given this history, she undergoes genetic testing and is found to have a mutation in the menin gene on the long arm of chromosome 11 (11q13). What is the most likely initial clinical manifestation for persons with this syndrome? A gastrinoma B Hirschsprung disease C hyperparathyroidism D medullary thyroid cancer E pituitary adenoma

The Correct Answer is: C This genetic mutation is found in patients with MEN1 syndrome, formerly known as Wermer Syndrome. About two-thirds of patients with this syndrome present with hyperparathyroidism. Gastrinomas (A) occur in about one-third and pituitary adenomas (E) in about half. Hirschsprung disease (B) and medullary thyroid carcinomas (D) are found in MEN2a syndrome.

A 45-year-old patient presents 2 days postoperatively with a partial thyroidectomy. She has been experiencing vomiting with diarrhea. On physical exam, her temperature is 101°F and jaundice is noted. Her heart rate is irregularly irregular with a rate of 200 bpm. What would be the most appropriate pharmacological intervention? A radioactive iodine (131I) B propranolol 80 mg C PTU 600 mg D iopanoic acid 500 mg

The Correct Answer is: C This patient is in a thyrotoxic crisis or thyroid storm. She needs to be admitted for monitoring and supportive care. The initial treatment would be PTU 600 mg loading dose followed by 200 to 300 mg every 6 hours given either by nasogastric tube or rectally.

An 18-year-old woman comes in for evaluation of "losing weight without meaning to." She also feels weak and in "always in the bathroom." Her appetite is normal but she "can't get enough to drink." Examination shows that she has lost 17# since her last visit a year ago. She is 66" tall and now weighs 120#. She is mildly orthostatic, but no other abnormalities are noted. A random blood sugar done in the office is 260mg/dl. Which results are most likely on measurement of her lipoproteins at this time? A decreased high density lipoprotein level B extremely elevated triglycerides C markedly increased total cholesterol D mildly elevated low density lipoproteins E normal profile

The Correct Answer is: D In persons with diabetes mellitus, type 1, low density lipoproteins, trigycerides (B), and total cholesterol (C) are likely to be slightly elevatated. High density lipoproteins (A) remain about the same as the patient's baseline. Once the glucose level is controlled, the lipid levels (E) on the profile typically return to normal.

When used for advanced carcinoma of the prostate, chronic administration of leuprolide inhibits the synthesis of androgens by _____________________ A blocking gonadotropin-releasing hormone (GnRH) receptors at the anterior pituitary B blocking luteinizing hormone (LH) receptors on interstitial (Leydig) cells of the testes C increasing the secretion of GnRH from the hypothalamus D inhibiting pulsatile secretion of gonadotropins from the anterior pituitary E upregulation of the number of GnRH receptors at the anterior pituitary

The Correct Answer is: D Leuprolide is a GnRH (LHRH) agonist that suppresses the pulsatile secretion of follicle stimulating hormone (FSH) and LH (gonadotropins) from the anterior pituitary when given chronically. Continuous administration of a GnRH agonist causes down-regulation of GnRH receptors on gonadotropes, which, in turn suppresses gonadotropin release and gonadal function. Decreased amounts of LH, in particular, lead to diminished production of androgens by the testes (especially DHT), which support prostate growth. It is believed that by interrupting the hormonal pathways that modulate prostatic growth, tumor development and metastasis is slowed

A 53-year-old man is taking a proton pump inhibitor for GERD symptoms, a beta blocker and a thiazide diuretic for hypertension, an SSRI for depression, and an over-the-counter NSAID as needed for aches and pains. He has developed gynecomastia and laboratory studies reveal an elevated prolactin level. If his hyperprolactinemai is due to one of his medications, which is the most likely cause? A the beta blocker B the NSAID C the proton pump inhibitor D the SSRI E the thiazide diuretic

The Correct Answer is: D Many medications cause hyperprolactinemia, including SSRIs, tricyclic antidepressants, and antipsychotics. Hydralazine and methyldopa, but not beta blockers (A), may also raise prolactin levels; likewise opioids, but not NSAIDS (B).

Your 45-year-old diabetic on a statin develops myalgias and you elect to check a creatine phosphokinase (CPK). While a normal CPK is 150 U/L, your patient has an elevated CPK. At what point must you discontinue the use of the statin? A > 150 U/L B > 300 U/L C > 450 U/L D > 1500 U/L E none of the above

The Correct Answer is: D Minor increases in creatine kinase (CK) activity in plasma are observed in some patients receiving reductase inhibitors, frequently associated with heavy physical activity. Rarely, patients may have marked elevations in CK activity, often accompanied by generalized discomfort or weakness in skeletal muscles. In all patients, CK should be measured at baseline. If muscle pain, tenderness, or weakness appears, CK should be measured immediately and the drug discontinued if activity is elevated significantly over baseline (> 1500 U/L). The myopathy usually reverses promptly upon cessation of therapy. If the association is unclear, the patient can be re-challenged under close surveillance. Myopathy in the absence of elevated CK has been reported.

5: A 28-year-old woman who was born and brought up just outside of Washington, DC, comes in for evaluation of vague "problems with swallowing." She has no other symptoms except "my neck is bigger than it used to be." Examination reveals only a diffuse, somewhat irregular, nontender enlargement of the thyroid gland with distinct masses palpable within it. What is the most likely diagnosis? A endemic goiter B Graves disease C Hashimoto thyroiditis D multinodular goiter E thyroid carcinoma

The Correct Answer is: D Multinodular goiter is the most likely in a woman with these findings in the United States. It may be nontoxic as in this case or toxic, i.e., producing excessive thyroid hormones which cause symptoms of hyperthyroidism. Endemic goiter (A), which may present as a simple enlargement of the thyroid or as a multinodular one, is found almost entirely in iodine-deficient areas of the world and is extremely rare in the U.S. In Graves disease (B), the thyroid is enlarged and may exhibit a thrill and a bruit. In addition, the patient would have other signs of hyperthyroidism. The thyroid in Hashimoto thyroiditis (C) is diffusely enlarged and firm with fine nodules. A thyroid carcinoma (E) usually presents as a firm, nontender nodule in the gland.

A 60-year-old man newly diagnosed with type 2 diabetes mellitus is most likely to have which of the following on ophthalmologic evaluation? A diabetic cataracts B glaucoma C nonproliferative retinopathy D normal fundi E proliferative retinopathy

The Correct Answer is: D Only about 20% of patients with type 2 diabetes have retinopathy at the time they are diagnosed. All patients with diabetes are at increased risk for premature development of cataracts (A) and glaucoma (B) occurs eventually in about 6% of diabetics. Nonproliferative retinopathy (C) is present in just under 2/3 of patients who have had type 2 diabetes for 16 years, and proliferative retinopathy (E) is far more common in patients with type 1

Your patient with type 2 diabetes mellitus (T2DM) who is already on metformin and pioglitazone but continues to have elevated fasting and postprandial blood sugars. Together, you elect to begin insulin glargine at 10 U QHS initially and have the patient self manage their upwards titration adding 1 IU SQ in the evening every day that their morning fasting glucose is above 100 mg/dL. You caution the patient that she should expect to take approximately how long to achieve well-controlled sugars? A immediately B about one week C about two weeks D about one month E about three months

The Correct Answer is: D There are many regimens to begin and titrate basal insulins. Regardless, most patients require approximately 45 units of glargine to reach therapeutic goals. If they began glargine at 10 IU and add 1 IU daily until they reach 45 IU, this should take approximately 35 more days, or approximately one month.

A new patient to the practice reports that his paternal grandfather had a tumor of the pituitary gland, his father had hyperparathyroidism, one uncle had a pancreatic tumor and another a thyroid cancer. His first cousin has "some kind of facial tumors" and another relative had treatment for "producing too much stomach acid." Given this family history, he should be screened for which of the following? A Carney complex B Cowden disease C McCune-Albright syndrome D MEN1 E Sipple syndrome

The Correct Answer is: D This family history strongly suggests Multiple Endocrine Neoplasia 1, which are inherited in an autosomal dominant manner and involve mutations on the long arm of chromosome 11 (11a13). Carney complex (A) is another syndrome of multiple endocrine neoplasia (MEN) with tumors of the adrenal cortex, pituitary gland, thyroid, and gonads and with hyperpigmentation and cardiac myxomas. Cowden disease (B) is also a syndrome of MEN including thyroid abnormalities, breast cancer and hamartomas. Another MEN, McCune-Albright syndrome (C) is associated with precocious puberty, Cushing syndrome, hyperthyroidism, and acromegaly. Sipple syndrome (E), also known as MEN2a. may develop medullary thyroid carcinoma, pheochromocytomas, or Hirschsprung disease.

A 63-year-old woman presents with shortness of breath, cough, and proximal muscle weakness of 1-month duration. On clinical exam, she is noted to have a blood pressure of 156/102 mm Hg, facial flushing, mild hirsutism, truncal obesity, marked proximal muscle weakness of both the upper and lower extremity, and hyperpigmentation over the palms and back of the neck. Laboratory exam reveals hypercortisolism and increased ACTH. Which of the following would be the most likely primary diagnosis in this patient? A lymphoma B ovarian cancer C renal cell carcinoma D small cell lung carcinoma

The Correct Answer is: D Tumor cells may secrete hormones that have the same biologic actions as the normal hormone. This patient's symptoms are consistent with adrenocorticoid hyperfunction. The most common cause of ectopic ACTH syndrome is small cell lung carcinoma. This should be suspected in any patient with risk factors for lung cancer.

Which of the following statements regarding diabetic medications is/are most correct? A Incretin-mimetics like exenatide commonly result in weight loss. B Metformin should be held prior to and for 48 hours after administration of ionidated contrast material. C Sulfonylureas have the highest risk of hypoglycemia of all oral diabetic agents. D Thiazolidinediones (TZDs) help preserve beta-cell function.

The Correct Answer is: E Incretin-mimetics act upon the satiety center of the brain and as a result promote weight loss, averaging approximately six pounds. Metformin may cause fatal lactic acidosis when given to patients on concomitantly-administered iodinated contrast dye. Sulfanylureas, in part because of first-pass metabolism, have the highest risk of hypoglycemia of all the oral anti-diabetic agents. TZDs have been found to preserve beta-cell function more so than any other agent.

A 35-year-old male presents complaining of increasingly constant headaches, double vision centrally, and a progressive loss of peripheral vision for two weeks. He has no previous headache history and denies any other medical conditions. Physical examination reveals bitemporal hemianopsia without additional neurologic findings. What is the most likely diagnosis? A Acute ischemic stroke B Circle of Willis ruptured aneurysm C Migraine headache D Multiple sclerosis E Pituitary adenoma

The Correct Answer is: E Pituitary adenomas, benign neoplasms associated with pituitary hormone secretory changes, may enlarge and become symptomatic. Symptoms are based upon the location and size of the tumor, and may include bitemporal hemianopsia, double vision, color desaturation, and visual acuity loss. Headaches may occur, due to associated pressure changes within the intrasellar space. Additional evaluation should include a T1-weighted MRI, screening laboratory tests, and a full ophthalmologic evaluation. These tests will also help evaluate for potential differential diagnoses, such as those listed. The patient's history is not consistent with an acute ischemic stroke or migraine headache. Although an unruptured aneurysm may have very similar findings to a pituitary tumor, ruptured aneurysms present with acute headache, nausea, vomiting, and potential changes in consciousness. Multiple sclerosis (MS) should remain on the differential for this patient and will also be evaluated through MRI (although the current findings are more consistent with a pituitary adenoma), and additional neurologic findings would be likely with MS.

A 62-year-old male diabetic patient with a past medical history of percutaneous transluminal coronary angioplasty (PTCA) with drug-eluting stent presents to review his blood work. He currently takes rosuvastatin 10 mg daily, metformin 1,000 mg BID, an 81 mg aspirin, and ramapril 5 mg, along with metoprolol ER 50 mg daily. His total cholesterol is 212 mg/dL, LDL-C is 126 mg/dL, HDL-C is 46 mg/dL, and triglycerides at 145 mg/dL. Of the following, what is the best choice for him to reach ATP-III revised goals? A Add a fenofibrate 145 mg daily. B Add garlic to his diet. C Add an omega-3 ethyl acid ester daily. D Increase his rosuvastatin to 20 mg daily. E Increase his rosuvastatin to 20 mg daily and add ezetimibe 10 mg daily.

The Correct Answer is: E Statins lower LDL-C by 20% to 55%, depending on the dose and statin used. Regardless, analysis of dose-response relationships for all statins demonstrates that the efficacy of LDL-C lowering is log-linear; LDL-C is reduced by 6% (from baseline) with each doubling of the dose. In doubling his rosuvastatin, it would be reasonable to expect his LDL-C to drop from 126 to 119 mg/dl, and this would not be a goal for the patient. While there is no outcome data, laboratory data would indicate that if ezetimibe were then added, LDL-C would drop approximately another 25% to approximately 89 mg/dl, closer to his ATP-III goal of 70 mg/dl. Intensifying his life style changes might then allow him to reach goal.

A 42-year-old woman has a developed a solid and quite firm thyroid mass that is approximately 1 cm diameter by palpation. She does not have any hoarseness, difficulty breathing or swallowing, or symptoms of thyroid disease. She has never been exposed to radiation and has no history of cancer, nor does she have a family history of thyroid or other cancers. A thyroid panel is within normal limits. What is the next step in the care of this patient? A fine-needle aspiration biopsy of the nodule B observation to see if the nodule increases in size C radionuclide thyroid scan D resection of the nodule E ultrasonography of the neck

The Correct Answer is: E The next step in the care of this woman is neck ultrasonography to determine its size and characteristics; this technique is preferred to CT or MRI scanning because it is accurate, noninvasive, far less costly. Should the ultrasound results be suspicious for malignancy, a fine-needle aspiration biopsy (A) should follow. A thyroid mass 1 cm in diameter or larger must be evaluated rather than observed (B). Radionuclide scanning (C) would be indicated if the patient's TSH were low but is otherwise limited in determining whether a thyroid mass is malignant. Resection (D) would be inappropriate until further testing is done

A 49-year-old man presents to the office complaining of general malaise with muscle aches, anorexia, fever, and severe pain over his anterior neck radiating to his ears. He states that he was ill about 2 weeks ago with a sore throat, but it resolved within a few days. On palpation, the thyroid gland is enlarged and tender. His laboratory workup shows a high T4 level and increased erythrocyte sedimentation rate (ESR). What is the most appropriate therapy for this patient's disease? A levothyroxine sodium B PTU therapy C radioiodine ablation D surgery E supportive therapy only

The Correct Answer is: E This is subacute, painful thyroiditis. This is a self-limiting disorder that at most requires symptomatic therapy. In mild cases, analgesics (ASA) are sufficient for pain relief and to decrease the inflammation. Prednisone may bring more relief if needed. Transient hypothyroidism should be treated as well.

29-year-old woman presents in July to your office with symptoms of palpitations, sore neck, and excessive sweating, despite using her air conditioner all the time. No surgical or trauma history is noted. She is currently not taking any medications. Vitals include the following: BP = 124/68, pulse = 110 beats per minute, respirations = 18 per minute, and temperature = 101 o F orally. Upon exam, her thyroid is mildly enlarged without nodules, and severely tender. No local erythema or heat is noted. What is the most likely diagnosis in this patient? A Thyroid abscess B Papillary thyroid cancer C Hashimoto's thyroiditis D Graves' disease E Subacute thyroiditis

The Correct Answer is: E This patient is presenting with signs and symptoms of hyperthyroidism, most likely caused by subacute thyroiditis. The enlarged and tender thyroid, along with the fever, are classic findings in this condition. Choice A, thyroid abscess, is a localized infection of the thyroid. It would present with heat, swelling, and erythema in the area of tenderness. Choice B, papillary thyroid cancer, commonly presents with either a local non-tender thyroid nodule or as lymph node metastasis. Choice C, Hashimoto's thyroiditis, is a cause of chronic thyroiditis and is associated with a clinical presentation of hypothyroidism

A 42-year-old woman comes in with a "lump in her neck." When she was a teenager, she underwent radiation treatment for "some kind of tumor" in her neck. Ultrasound reveals a 1.5 cm lesion in the left lobe of the thyroid; biopsy confirms papillary carcinoma. Thorough evaluation reveals no evidence of metastasis. Which of the following is the initial treatment of choice for her? A chemotherapy B lobectomy with isthmectomy C radiation therapy D radioactive iodine therapy E total thyroidectomy

The Correct Answer is: E Total or near-total thyroidectomy is indicated for this woman. No chemotherapy (A) is available for thyroid cancer. Lobectomy with isthmectomy is reserved for papillary carcinoma that is less than 1 cm in size in persons with no history of radiation exposure and no evidence of metastasis. Radiation therapy (C) is used to treat bone metastasis and anaplastic carcinoma. Radioactive iodine therapy (D) may be used following thyroidectomy to ablate any remnant of the gland and to treat cancer that has metastasized or is otherwise high risk.

A patient with long-standing, untreated acromegaly is seen in your office with symptoms of severe headaches. After completing a thorough history and physical exam, you order a set of x-rays including a skull series. Which of the following findings would you expect in this patient? A Punched out lesions B Basilar skull fracture C Metastatic bone lesions D Enlarged sella turcica E Thinning of the skull

The correct choice is (D), enlarged sella turcica. This finding is seen in 90% of patients with acromegaly. Other findings on skull radiographs include thickened calvarium (upper portion of the skull), enlarged mandible, and sinuses. Bony growth is a hallmark of the disease. The pituitary adenoma, which typically causes the disease, can be found in the sella turcica. This disorder doesn't typically metastasize, and is not associated with metastatic bone cancer, as noted in choice C. Choice A, punched out lesions, are commonly associated with Paget's disease. There is no history of head trauma, as would be the case in choice B, basilar skull fracture. As noted earlier, the skull may be thickened. Therefore, choice E, thinning of the skull, would not fit this patient's presentation

A 40-year-old female presents to the clinic with symptoms of a 10 pound unintentional weight loss, diarrhea, and palpitations. She has a positive family history for thyroid disease and diabetes mellitus. Her screening TSH level is 0.15mIU/L, and her fasting plasma glucose is 105mg/dL. Which physical exam finding would you expect to find in this patient? A Tremor B Thin hair C Puffy face D Delayed reflexes E Bradycardia

The correct choice is A, tremor. This question first requires the identification of hyperthyroidism in the patient. The symptoms of weight loss, diarrhea, and palpitations, with a low TSH level, are characteristic of the disorder. Hyperthyroidism also presents most commonly in women in the third and fourth decade, as well as in the elderly. Common physical exam findings of hyperthyroidism include tachycardia, warm moist skin, tremor, goiter, muscle weakness, lid retraction or lag, and gynecomastia

What is the definitive treatment for the majority of patients presenting with mild symptoms of hyperthyroidism secondary to subacute thyroiditis? A Subtotal thyroidectomy B Oral methimazole C Symptomatic treatment D Radioactive iodine E Antibiotics

The correct choice is C, symptomatic treatment. Most patients with subacute thyroiditis and symptoms of hyperthyroidism require only symptomatic treatment, with non-steroidal anti-inflammatory medications and/or beta blockers, for any cardiac symptoms including palpitations and tachycardia. Occasionally, patients may require a course of prednisone for this acute inflammatory condition. Most patients will recover spontaneously within a few months. Choices A, B, D, and E are not necessary in this condition. Most cases of subacute thyroiditis are associated with viral infections, and resolve without additional thyroid medications.

A young man presents with difficulty breathing at times. Upon exam you note evidence of a firm, fixed thyroid nodule with extension toward the trachea and surrounding muscles. The patient has a family history of thyroid cancer. You are concerned that the patient may have medullary thyroid cancer. After you confirm your suspicion, which of the following genetic tests would you suggest to his family members? A BRCA 1 B APC C CFTR D ret proto-oncogene E CAG triplet expansion

The correct choice is D, ret proto-oncogene. Genetic testing to identify this oncogene is available for patients whom you suspect have medullary thyroid cancer and/or pheochromocytoma, and for their asymptomatic families who may be at risk for the same disorders. A mutation in this gene leads to uncontrolled growth of the C cells of the thyroid. Choice A, BRCA 1, is an oncogene that is associated with risk for breast, ovarian, colon, and prostate cancers. Choice B, APC, is an oncogene noted in the familial colon cancer syndrome known as familial adenomatous polyposis. DNA analysis for CFTR mutations, choice C, can identify the gene mutations in patients with cystic fibrosis in the vast majority of instances. Choice E, CAG triplet expansion, is the result of a gene mutation in Huntington's disease.

Consideration should be given to screening patients with Type 1 Diabetes Mellitus for which of the following disorders? A sarcoidosis B Sheehan's syndrome C Sjögren's Syndrome D thyroid disease

Type 1 diabetes mellitus (T1DM) is an autoimmune disease. As such, patients have a significantly higher risk of other autoimmune diseases, including celiac and thyroid disease.


Set pelajaran terkait

NUR1025 Most Missed Questions Exam 2

View Set

IXL compound complex simple and compound complex sentences

View Set

Georgia 25 Hour Post Licencess Class Lead based paint

View Set

Chapter 10: Professional Development

View Set

Campbell Biology Chapter 7 Membranes

View Set

Third-Party Beneficiaries; Assignment & Delegation; Novation

View Set